Fisica, Fisica Matematica, Fisica applicata, Astronomia
Discussioni su argomenti di Fisica, Fisica Matematica, Astronomia e applicazioni della Fisica
Domande e risposte
Ordina per
In evidenza

tre cariche puntiformi sono disposte ai vertici di un triangolo equilatero di lato d=10cm:
$ q_1=(-d/2,0)=-q;q_2=(d/2,0)=-q;q_3=(0,d)=2q $
calcolare il potenziale elettrico V nel punto P di coordinate x=0; y=40cm
sia con l'approssimazione di dipolo, sia con il principio di sovrapposizione.
quello che non mi è chiaro in questo esercizio è la geometria:
per calcolare il potenziale nell'approssimazione di dipolo, scrivo $ V(P)=1/{4πε_0}p/r^2 $
non riesco a capire perchè r debba essere $ r=(y_0-d/2(√3)/2) $
il mio secondo ...
Salve a tutti, stavo svolgendo qualche esercizio di Dinamica sul libro "Mazzoldi,Saggion,Voci" in vista dell'esame di Fisica Generale. In particolare mi sto trovando in difficoltà su un problema
Per quanto riguarda la parte del trovare la velocità non ho avuto particolari problemi e mi torna come dovrebbe tornare, cioè $\upsilon(\theta)=sqrt(2*g*R*cos(\theta)$
Dunque dato che la velocità non è costante e il moto è circolare allora so che l'accelerazione è formata da due componenti, una tangente e ...

Ho svolto questo esercizio, ma non riesco a comprendere le note del mio professore riguardanti il punto 3
Io ho semplicemente immaginanto che collegando le due sferette, essendo identiche la carica si sarebbe disposta in modo identico sulle due, ho quindi sommato le cariche e diviso per due per trovare la carica risultante dal collegamento e poi impostato che la funzione potenziale (che è scalare) nel punto sulla superficie del corpo A rispettasse:
$Q/(4pi\epsilon_0r_s)+Q/(4pi\epsilon_0d$ è il ...

buonasera, sto trovando alcune difficoltà su questi $3$ esercizi.
1) in un punto a $2cm$ dall'asse di un solenoide di raggio $r=3cm$ e $800 "spire"/m$ viene indotto un campo elettrico di $4 muV/m$. A quale velocità sta cambiando la corrente nel solenoide in quell'istante? [esprimere la soluzione in $A/s$]
Purtroppo su questo esercizio non ho per nulla idea su come procedere. Infatti non mi è neanche chiara quale sia la richiesta per ...

Ho un dubbio su una pagina del mio libro che è consigliato dal Professore.
Non mi ci ritrovo con l'affermazione di prendere corpi interni cairchi $Q_1$... $Q_n$ e poi collegandoli all'involucro a potenziale $V_0$ considerare che quei corpi mantengano carica $Q_i$ prestabilita. Questo non mi sembra per nulla vero, perché collegandoli la carica potrebbe (e dovrebbe) anche ripartirsi sull'involucro conduttore, considerazione che ...

Si consideri un oscillatore di massa $m$ e pulsazione $w$. Usando esclusivamente il principio di Heisenberg e ragioni di simmetria, stimare:
a)l'energia dello stato fondamentale;
b) la deviazione standard di x;
c) il valore atteso di P;
d) la deviazione standard di P;
per quanto riguarda il punto a so che $E_n=h/(2pi)w(n+1/2)$ che per $n=0$ ottengo $E_n=h/(4pi)w$
siccome l'esercizio mi dice di utilizzare solo heinseberg io ho pensato di rispondere al ...

Salve a tutti, non riesco a capire come mai non imposto bene il seguente, esercizio, non capisco dove sbaglio.
Due punti materiali di uguale massa (12.6kg) si muovono in direzioni opposte e hanno velocità iniziali pari a 22.4 m/s e 7.25 m/s. I due punti materiali si urtano in modo anaelastico, viene richiesto di calcolare la variazione di energia cinetica del sistema nell'ipotesi che la velocità relativa tra i due corpi dopo l'urto sia la metà della velocità relativa tra i due corpi all'inizio. ...

salve! ho un dubbio su come calcolare la velocità. Avevo pensato la conservazione dell'energia considerando che Ki=0 e Uf=0 perchè raggiunge la quota che ho scelto come riferimento ma viene energia cinetica negativa quindi non credo vada bene.
Questo procedimento è corretto oppure devo procedere diversamente?

Si consideri la disuguaglianza di Clausius. E' chiaro che se il ciclo e' reversibile allora vale l'uguaglianza. E' vero che l'uguaglianza vale solo nel caso reversibile?
La stessa questione e' affrontata nel link riportato sotto, ma senza risposte convincenti.
https://physics.stackexchange.com/questions/518206/how-does-fermi-jump-to-this-conclusion-in-clausius-inequality?noredirect=1&lq=1

Ciao, vorrei cercare di chiarire un dubbio su versori polari.
Ho compreso che le coordinate polari sono dei parametri che mi indicano la posizione del punto con una dupla (cioè una coppia di parametri/coordinate che indicano la distanza dall'origine e l'angolo di apertura rispetto all'asse iniziale).
Il dubbio è invece correlato ai versori polari, cosa sono? Non riesco a concepirli nel senso che sono dei vettori diretti uno come la direzione radialeel'altro tangente al movimento (arco) creato ...

Un cubetto di ferro subisce una spinta di 145N quando è completamente immerso in acqua. Quanto misura lo spigolo del cubo?
(24,5cm)
avendo la spinta $S=145N$ e la densità dell'acqua $d=1000kg / m^3$ come bisogna procedere? Devo calcolare il volume? non mi è chiarissimo questo argomento

Ragazzi i diodi raddrizzatori trasformano una corrente alternata in continua. Ma se io volevo fare la stessa operazione ma senza l uso di diodi fisici, in che altra modo ci potrei riuscire in wireless ??

Avrei un esercizio che mi blocca nell'ultimo punto:
Su una superficie sferica di raggioR= 10 cm, nel vuoto, è uniformemente distribuita una caricaelettrica Q negativa. Uno ione carbonio (ionizzato una sola volta, ovvero avente la carica del proto-ne), posto inizialmente al centro della sfera, viene lanciato radialmente verso un foro praticato sullasuperficie (abbastanza piccolo da poterne trascurare gli effetti sul campo elettrico). Si osserva che loione riesce ad allontanarsi, sfuggendo ...

Ciao , mi sa che mi sono del tutto arenato su questo esercizio:
Due strati piani infiniti, di materiale isolante e di spessorebsono disposti lungo l’asse verticale e accostati l’uno all’altro. Supponendo che la loro densità di carica sia rispettivamente +ρ e−ρ, determinare l’andamento del campo elettrico in ogni punto dello spazio.
Non ho idea di come impostarlo, ho provato con gauss e vari flussi ma non mi viene nulla di utile
Qualche idea?

Sto studiano la parte del magnetismo e ho finito una prima lettura. Ho però un dubbio che mi preme capire e provo ad esporvi.
Sebbene abbia capito l'utilità dell'introdurre spire infinitesime per descrivere il magnetismo (e il correlato campo B), a conti fatti non capisco bene però come sfruttare il vettore magnetizzazione e B stessi nel calcolo di una forza magnetica.
Se per la parte di elettricità è semplice: dato il campo E ci piazzo un bel q davanti: qE e trovo la forza elettrica.
Ma per ...

Sera a tutti voi. Ho un dubbio che mi è sorto svolgendo un esercizio di un cilindro carico infinitamente esteso e dove chiedeva di calcolare campo e potenziale ad esso associati.
Il punto che mi ha creato dubbi è il fatto che il potenziale a infinito mi diverge. E' quindi sensato poter porre un valore di potenziale nullo a infinito venendomi a trovare con $V(x)-V(oo)=oo$ quindi è un po' insensato scrivere $V(x)=oo+V(oo)$ primo perché infinito non è un punto quindi come definisco una ...

Ci sono due relazioni vettoriali che ho affrontato e che non capisco pienamente. E soprattutto non mi restano in testa, forse proprio perché non capite. Vorrei sapere come come le ricordate/ricavate :
- $V\nabla^2V=\vec\nabla*(V\vec\nablaV)-(\vec\nabla^2V)$
- $\vec\nablaxx(\vec\nablaxx\vecA)=\vec\nabla(\vec\nabla*\vecA)-\nabla^2\vecA$
Spero in qualche spunto,
Grazie!

Buonasera a tutti,
qualcuno saprebbe spiegarmi perché il determinante della matrice di rotazione (la matrice 3x3 costituita dai coseni direttori di un corpo rigido) è pari ad 1?
Grazie in anticipo!

Buonasera, ho il seguente esercizio:
Un cavo coassiale è costituito da un conduttore pieno cilindrico di raggio R1 e da una guaina cilindrica conduttrice di spessore trascurabile di raggio R2 tale che R1 < R2, percorsi dalla stessa corrente I in versi opposti. Il modulo del campo magnetico risulta
• $B = (mu_0I)/(2pir)$ ovunque (r e’ la distanza dall’asse del cilindro)
• B = 0 per r < R1
• B = 0 per r > R2
Secondo me la risposta corretta è B= 0 per r > R2, ho fatto il seguente ragionamento, ...

Sto studiando sul Mazzoldi fisica volume II (quello grande).
Capitolo sui conduttori.
Campo all'interno del conduttore nullo, E=0. Adesso prendo un elettrone e lo inserisco al centro del conduttore(suppongo un conduttore cubico). Questo per quanto detto non è soggetto ad alcuna forza e quindi rimane fermo al centro. Ora però non ho più E=0 e ne tanto meno questo si va a posizionare in superficie.
Se ne metto 2 di elettroni è chiaro il fatto che questi si vanno a posizionare in superficie, ma ...